Abstract algebra subgroup proof verificationSmallest subgroup containing a subset $A$Proof Verification (Set...

Is the UK legally prevented from having another referendum on Brexit?

Reason for small-valued feedback resistors in low noise Op Amp

Isn't a semicolon (';') needed after a function declaration in C++?

What happens if both players misunderstand the game state until it's too late?

Minimum Viable Product for RTS game?

What sort of grammatical construct is ‘Quod per sortem sternit fortem’?

Can you say "leftside right"?

How can I give a Ranger advantage on a check due to Favored Enemy without spoiling the story for the player?

Can a Way of Shadow Monk use Shadow Step to teleport to a dark ceiling and then body slam another creature?

What does "don't have a baby" imply or mean in this sentence?

Why is Shelob considered evil?

Why write a book when there's a movie in my head?

Is it possible to narrate a novel in a faux-historical style without alienating the reader?

Taking an academic pseudonym?

Converting numbers to words - Python

Boss asked me to sign a resignation paper without a date on it along with my new contract

How to transport 10,000 terrestrial trolls across ocean fast?

How can I prep for the Curse of Strahd adventure effectively?

What is an explicit bijection in combinatorics?

Integer but not Laurent sequences

Multiple null checks in Java 8

How do I narratively explain how in-game circumstances do not mechanically allow a PC to instantly kill an NPC?

In the Lost in Space intro why was Dr. Smith actor listed as a special guest star?

Have any astronauts or cosmonauts died in space?



Abstract algebra subgroup proof verification


Smallest subgroup containing a subset $A$Proof Verification (Set Theory)Permutation(?) mappingCritique of this subgroup proof?Convincing normal subgroup proof?how is my proof on composition functionsProof verification: Every group of order n! contains non trivial normal subgroup (without Sylow)If $g circ f$ is injective and $f$ is surjective, then $g$ is injective.Julius König's proof of Schröder–Bernstein theoremSubgroup of a finite group $H$ and $G$













8












$begingroup$


This is my first attempt at a formally written proof so I would appreciate any pointers as far as proof-writing technique or the validity of the actual proof itself.



Note: I have not formally taken abstract algebra or a proof-writing course so I am sure that I am lacking in many proof-writing aspects, so I would really love a lot of constructive criticism both on the actual proof itself, and the way I wrote the proof. I also go into greater detail than might be appropriate for this type of proof because I am shaky on a lot of the math foundations so I figure that any imperfections in my knowledge will be more easily seen with a more explicit construction of this proof. Thank you all in advance.




Let $G$ be a finite group, and let $S$ be a nonempty subset of $G$. Suppose $S$ is closed with respect to multiplication. Prove that $S$ is a subgroup of $G$. (HINT: It remains to prove that $S$ contains $e$ and is closed with respect to inverses. Let $S$ = {$a_1$ ... $a_n$}. If $a_i$ $∈$ $S$, consider the distinct elements $a_ia_1$, $a_ia_2$, $...$ $a_ia_n$




Proof:



First we will define a function $A_1 : S rightarrow S$ that maps $s mapsto a_1s$. This function is injective because $$a_1y = a_1x$$ $$a^{-1}_1a^1y = a^{-1}_1a_1x$$ $$y = x$$



The function is then surjective because $A_1(S) subseteq S$ and since $A_1$ is injective, it contains $|S|$ elements. Therefore $A_1$ maps onto every element in $S$ and is therefore surjective as well.



This means that $a_1$ is in the image of $A_1$. Therefore $$A_1(a_1) = a_1$$ $$a_1s = a_1$$ $$s = e$$



Since $S$ is closed under multiplication, $e in S$.



Next, we will define a function $A_2 : S rightarrow S$ that maps $s mapsto a^2_1s$. This function is also injective $$a^2_1x = a^2_1y$$ $$x = y$$



It follows that this function is also surjective since it too is injective and contains |S| elements.



This means that $a_1$ is in the image of $A_2$ as well. Therefore $$A_2(z) = a_1$$ $$a^2_1z = a_1$$ $$z = a^{-1}_1$$



Since $S$ is closed under multiplication $a^{-1}_1 in S$.



Therefore $e, a^{-1}_1 in S$ so $S$ is a subgroup of $G$.



Please tear this apart! Thanks in advance.










share|cite|improve this question







New contributor




ForIgreaterthanJ is a new contributor to this site. Take care in asking for clarification, commenting, and answering.
Check out our Code of Conduct.







$endgroup$

















    8












    $begingroup$


    This is my first attempt at a formally written proof so I would appreciate any pointers as far as proof-writing technique or the validity of the actual proof itself.



    Note: I have not formally taken abstract algebra or a proof-writing course so I am sure that I am lacking in many proof-writing aspects, so I would really love a lot of constructive criticism both on the actual proof itself, and the way I wrote the proof. I also go into greater detail than might be appropriate for this type of proof because I am shaky on a lot of the math foundations so I figure that any imperfections in my knowledge will be more easily seen with a more explicit construction of this proof. Thank you all in advance.




    Let $G$ be a finite group, and let $S$ be a nonempty subset of $G$. Suppose $S$ is closed with respect to multiplication. Prove that $S$ is a subgroup of $G$. (HINT: It remains to prove that $S$ contains $e$ and is closed with respect to inverses. Let $S$ = {$a_1$ ... $a_n$}. If $a_i$ $∈$ $S$, consider the distinct elements $a_ia_1$, $a_ia_2$, $...$ $a_ia_n$




    Proof:



    First we will define a function $A_1 : S rightarrow S$ that maps $s mapsto a_1s$. This function is injective because $$a_1y = a_1x$$ $$a^{-1}_1a^1y = a^{-1}_1a_1x$$ $$y = x$$



    The function is then surjective because $A_1(S) subseteq S$ and since $A_1$ is injective, it contains $|S|$ elements. Therefore $A_1$ maps onto every element in $S$ and is therefore surjective as well.



    This means that $a_1$ is in the image of $A_1$. Therefore $$A_1(a_1) = a_1$$ $$a_1s = a_1$$ $$s = e$$



    Since $S$ is closed under multiplication, $e in S$.



    Next, we will define a function $A_2 : S rightarrow S$ that maps $s mapsto a^2_1s$. This function is also injective $$a^2_1x = a^2_1y$$ $$x = y$$



    It follows that this function is also surjective since it too is injective and contains |S| elements.



    This means that $a_1$ is in the image of $A_2$ as well. Therefore $$A_2(z) = a_1$$ $$a^2_1z = a_1$$ $$z = a^{-1}_1$$



    Since $S$ is closed under multiplication $a^{-1}_1 in S$.



    Therefore $e, a^{-1}_1 in S$ so $S$ is a subgroup of $G$.



    Please tear this apart! Thanks in advance.










    share|cite|improve this question







    New contributor




    ForIgreaterthanJ is a new contributor to this site. Take care in asking for clarification, commenting, and answering.
    Check out our Code of Conduct.







    $endgroup$















      8












      8








      8


      1



      $begingroup$


      This is my first attempt at a formally written proof so I would appreciate any pointers as far as proof-writing technique or the validity of the actual proof itself.



      Note: I have not formally taken abstract algebra or a proof-writing course so I am sure that I am lacking in many proof-writing aspects, so I would really love a lot of constructive criticism both on the actual proof itself, and the way I wrote the proof. I also go into greater detail than might be appropriate for this type of proof because I am shaky on a lot of the math foundations so I figure that any imperfections in my knowledge will be more easily seen with a more explicit construction of this proof. Thank you all in advance.




      Let $G$ be a finite group, and let $S$ be a nonempty subset of $G$. Suppose $S$ is closed with respect to multiplication. Prove that $S$ is a subgroup of $G$. (HINT: It remains to prove that $S$ contains $e$ and is closed with respect to inverses. Let $S$ = {$a_1$ ... $a_n$}. If $a_i$ $∈$ $S$, consider the distinct elements $a_ia_1$, $a_ia_2$, $...$ $a_ia_n$




      Proof:



      First we will define a function $A_1 : S rightarrow S$ that maps $s mapsto a_1s$. This function is injective because $$a_1y = a_1x$$ $$a^{-1}_1a^1y = a^{-1}_1a_1x$$ $$y = x$$



      The function is then surjective because $A_1(S) subseteq S$ and since $A_1$ is injective, it contains $|S|$ elements. Therefore $A_1$ maps onto every element in $S$ and is therefore surjective as well.



      This means that $a_1$ is in the image of $A_1$. Therefore $$A_1(a_1) = a_1$$ $$a_1s = a_1$$ $$s = e$$



      Since $S$ is closed under multiplication, $e in S$.



      Next, we will define a function $A_2 : S rightarrow S$ that maps $s mapsto a^2_1s$. This function is also injective $$a^2_1x = a^2_1y$$ $$x = y$$



      It follows that this function is also surjective since it too is injective and contains |S| elements.



      This means that $a_1$ is in the image of $A_2$ as well. Therefore $$A_2(z) = a_1$$ $$a^2_1z = a_1$$ $$z = a^{-1}_1$$



      Since $S$ is closed under multiplication $a^{-1}_1 in S$.



      Therefore $e, a^{-1}_1 in S$ so $S$ is a subgroup of $G$.



      Please tear this apart! Thanks in advance.










      share|cite|improve this question







      New contributor




      ForIgreaterthanJ is a new contributor to this site. Take care in asking for clarification, commenting, and answering.
      Check out our Code of Conduct.







      $endgroup$




      This is my first attempt at a formally written proof so I would appreciate any pointers as far as proof-writing technique or the validity of the actual proof itself.



      Note: I have not formally taken abstract algebra or a proof-writing course so I am sure that I am lacking in many proof-writing aspects, so I would really love a lot of constructive criticism both on the actual proof itself, and the way I wrote the proof. I also go into greater detail than might be appropriate for this type of proof because I am shaky on a lot of the math foundations so I figure that any imperfections in my knowledge will be more easily seen with a more explicit construction of this proof. Thank you all in advance.




      Let $G$ be a finite group, and let $S$ be a nonempty subset of $G$. Suppose $S$ is closed with respect to multiplication. Prove that $S$ is a subgroup of $G$. (HINT: It remains to prove that $S$ contains $e$ and is closed with respect to inverses. Let $S$ = {$a_1$ ... $a_n$}. If $a_i$ $∈$ $S$, consider the distinct elements $a_ia_1$, $a_ia_2$, $...$ $a_ia_n$




      Proof:



      First we will define a function $A_1 : S rightarrow S$ that maps $s mapsto a_1s$. This function is injective because $$a_1y = a_1x$$ $$a^{-1}_1a^1y = a^{-1}_1a_1x$$ $$y = x$$



      The function is then surjective because $A_1(S) subseteq S$ and since $A_1$ is injective, it contains $|S|$ elements. Therefore $A_1$ maps onto every element in $S$ and is therefore surjective as well.



      This means that $a_1$ is in the image of $A_1$. Therefore $$A_1(a_1) = a_1$$ $$a_1s = a_1$$ $$s = e$$



      Since $S$ is closed under multiplication, $e in S$.



      Next, we will define a function $A_2 : S rightarrow S$ that maps $s mapsto a^2_1s$. This function is also injective $$a^2_1x = a^2_1y$$ $$x = y$$



      It follows that this function is also surjective since it too is injective and contains |S| elements.



      This means that $a_1$ is in the image of $A_2$ as well. Therefore $$A_2(z) = a_1$$ $$a^2_1z = a_1$$ $$z = a^{-1}_1$$



      Since $S$ is closed under multiplication $a^{-1}_1 in S$.



      Therefore $e, a^{-1}_1 in S$ so $S$ is a subgroup of $G$.



      Please tear this apart! Thanks in advance.







      abstract-algebra group-theory proof-verification proof-writing finite-groups






      share|cite|improve this question







      New contributor




      ForIgreaterthanJ is a new contributor to this site. Take care in asking for clarification, commenting, and answering.
      Check out our Code of Conduct.











      share|cite|improve this question







      New contributor




      ForIgreaterthanJ is a new contributor to this site. Take care in asking for clarification, commenting, and answering.
      Check out our Code of Conduct.









      share|cite|improve this question




      share|cite|improve this question






      New contributor




      ForIgreaterthanJ is a new contributor to this site. Take care in asking for clarification, commenting, and answering.
      Check out our Code of Conduct.









      asked 5 hours ago









      ForIgreaterthanJForIgreaterthanJ

      783




      783




      New contributor




      ForIgreaterthanJ is a new contributor to this site. Take care in asking for clarification, commenting, and answering.
      Check out our Code of Conduct.





      New contributor





      ForIgreaterthanJ is a new contributor to this site. Take care in asking for clarification, commenting, and answering.
      Check out our Code of Conduct.






      ForIgreaterthanJ is a new contributor to this site. Take care in asking for clarification, commenting, and answering.
      Check out our Code of Conduct.






















          2 Answers
          2






          active

          oldest

          votes


















          3












          $begingroup$

          You don't need to use the second map $A_2$. Once you know that $e in S$, you know by the first part of your proof that $A_1$ is surjective, so for some $s in S, a_1s=e$. Then $s=a^{-1}$.



          Another way to prove this result, by the way, is to just take all possible powers of $a_1$, which all are contained within $S$ because $S$ is multiplicatively closed. They have to repeat at some point because $S$ is finite. If $a^m=a^{m+k}=a^ma^k$, then $a^k=e in S$, and $aa^{k-1}=a^k=e Rightarrow a^{k-1}=a^{-1} in S$.



          As for proof-writing, one small nit. I don't think the line $A_1(a_1) = a_1$ conveys your intended meaning. I think you mean to say $a_1 in A_1(S)$.






          share|cite|improve this answer









          $endgroup$





















            2












            $begingroup$

            The proof presented here by our OP ForIgreaterthan_I appears to be logically valid; indeed, I like it because it is clever and innovative.



            The way I would approach this, which alas is standard and not in fact overly clever, being as it is indeed rather obvious and straightforward, is as follows:



            Let



            $s in S; tag 1$



            consider the powers



            $s^i in S; tag 2$



            since



            $S subset G tag 3$



            and



            $vert G vert < infty, tag 4$



            we have



            $vert S vert < infty tag 5$



            as well; thus the sequence



            $s, s^2, s^3, ldots, s^i, s^{i + 1}, ldots tag 6$



            must repeat itself at some point; that is,



            $exists k, l in Bbb N, ; l ge k + 1, tag 7$



            with



            $s^l = s^k; tag 8$



            then



            $s^{l - k} = e; tag 9$



            thus



            $e = s^{l - k} in S; tag{10}$



            it follows then that



            $s^{l - k - 1}s = e tag{11}$



            and clearly



            $s^{l - k - 1} in S; tag{12}$



            thus the group identity $e in S$, and every $s in S$ has an inverse in $S$; $S$ is thus a subgroup of $G$.






            share|cite|improve this answer









            $endgroup$













              Your Answer





              StackExchange.ifUsing("editor", function () {
              return StackExchange.using("mathjaxEditing", function () {
              StackExchange.MarkdownEditor.creationCallbacks.add(function (editor, postfix) {
              StackExchange.mathjaxEditing.prepareWmdForMathJax(editor, postfix, [["$", "$"], ["\\(","\\)"]]);
              });
              });
              }, "mathjax-editing");

              StackExchange.ready(function() {
              var channelOptions = {
              tags: "".split(" "),
              id: "69"
              };
              initTagRenderer("".split(" "), "".split(" "), channelOptions);

              StackExchange.using("externalEditor", function() {
              // Have to fire editor after snippets, if snippets enabled
              if (StackExchange.settings.snippets.snippetsEnabled) {
              StackExchange.using("snippets", function() {
              createEditor();
              });
              }
              else {
              createEditor();
              }
              });

              function createEditor() {
              StackExchange.prepareEditor({
              heartbeatType: 'answer',
              autoActivateHeartbeat: false,
              convertImagesToLinks: true,
              noModals: true,
              showLowRepImageUploadWarning: true,
              reputationToPostImages: 10,
              bindNavPrevention: true,
              postfix: "",
              imageUploader: {
              brandingHtml: "Powered by u003ca class="icon-imgur-white" href="https://imgur.com/"u003eu003c/au003e",
              contentPolicyHtml: "User contributions licensed under u003ca href="https://creativecommons.org/licenses/by-sa/3.0/"u003ecc by-sa 3.0 with attribution requiredu003c/au003e u003ca href="https://stackoverflow.com/legal/content-policy"u003e(content policy)u003c/au003e",
              allowUrls: true
              },
              noCode: true, onDemand: true,
              discardSelector: ".discard-answer"
              ,immediatelyShowMarkdownHelp:true
              });


              }
              });






              ForIgreaterthanJ is a new contributor. Be nice, and check out our Code of Conduct.










              draft saved

              draft discarded


















              StackExchange.ready(
              function () {
              StackExchange.openid.initPostLogin('.new-post-login', 'https%3a%2f%2fmath.stackexchange.com%2fquestions%2f3123182%2fabstract-algebra-subgroup-proof-verification%23new-answer', 'question_page');
              }
              );

              Post as a guest















              Required, but never shown

























              2 Answers
              2






              active

              oldest

              votes








              2 Answers
              2






              active

              oldest

              votes









              active

              oldest

              votes






              active

              oldest

              votes









              3












              $begingroup$

              You don't need to use the second map $A_2$. Once you know that $e in S$, you know by the first part of your proof that $A_1$ is surjective, so for some $s in S, a_1s=e$. Then $s=a^{-1}$.



              Another way to prove this result, by the way, is to just take all possible powers of $a_1$, which all are contained within $S$ because $S$ is multiplicatively closed. They have to repeat at some point because $S$ is finite. If $a^m=a^{m+k}=a^ma^k$, then $a^k=e in S$, and $aa^{k-1}=a^k=e Rightarrow a^{k-1}=a^{-1} in S$.



              As for proof-writing, one small nit. I don't think the line $A_1(a_1) = a_1$ conveys your intended meaning. I think you mean to say $a_1 in A_1(S)$.






              share|cite|improve this answer









              $endgroup$


















                3












                $begingroup$

                You don't need to use the second map $A_2$. Once you know that $e in S$, you know by the first part of your proof that $A_1$ is surjective, so for some $s in S, a_1s=e$. Then $s=a^{-1}$.



                Another way to prove this result, by the way, is to just take all possible powers of $a_1$, which all are contained within $S$ because $S$ is multiplicatively closed. They have to repeat at some point because $S$ is finite. If $a^m=a^{m+k}=a^ma^k$, then $a^k=e in S$, and $aa^{k-1}=a^k=e Rightarrow a^{k-1}=a^{-1} in S$.



                As for proof-writing, one small nit. I don't think the line $A_1(a_1) = a_1$ conveys your intended meaning. I think you mean to say $a_1 in A_1(S)$.






                share|cite|improve this answer









                $endgroup$
















                  3












                  3








                  3





                  $begingroup$

                  You don't need to use the second map $A_2$. Once you know that $e in S$, you know by the first part of your proof that $A_1$ is surjective, so for some $s in S, a_1s=e$. Then $s=a^{-1}$.



                  Another way to prove this result, by the way, is to just take all possible powers of $a_1$, which all are contained within $S$ because $S$ is multiplicatively closed. They have to repeat at some point because $S$ is finite. If $a^m=a^{m+k}=a^ma^k$, then $a^k=e in S$, and $aa^{k-1}=a^k=e Rightarrow a^{k-1}=a^{-1} in S$.



                  As for proof-writing, one small nit. I don't think the line $A_1(a_1) = a_1$ conveys your intended meaning. I think you mean to say $a_1 in A_1(S)$.






                  share|cite|improve this answer









                  $endgroup$



                  You don't need to use the second map $A_2$. Once you know that $e in S$, you know by the first part of your proof that $A_1$ is surjective, so for some $s in S, a_1s=e$. Then $s=a^{-1}$.



                  Another way to prove this result, by the way, is to just take all possible powers of $a_1$, which all are contained within $S$ because $S$ is multiplicatively closed. They have to repeat at some point because $S$ is finite. If $a^m=a^{m+k}=a^ma^k$, then $a^k=e in S$, and $aa^{k-1}=a^k=e Rightarrow a^{k-1}=a^{-1} in S$.



                  As for proof-writing, one small nit. I don't think the line $A_1(a_1) = a_1$ conveys your intended meaning. I think you mean to say $a_1 in A_1(S)$.







                  share|cite|improve this answer












                  share|cite|improve this answer



                  share|cite|improve this answer










                  answered 4 hours ago









                  Robert ShoreRobert Shore

                  1,47915




                  1,47915























                      2












                      $begingroup$

                      The proof presented here by our OP ForIgreaterthan_I appears to be logically valid; indeed, I like it because it is clever and innovative.



                      The way I would approach this, which alas is standard and not in fact overly clever, being as it is indeed rather obvious and straightforward, is as follows:



                      Let



                      $s in S; tag 1$



                      consider the powers



                      $s^i in S; tag 2$



                      since



                      $S subset G tag 3$



                      and



                      $vert G vert < infty, tag 4$



                      we have



                      $vert S vert < infty tag 5$



                      as well; thus the sequence



                      $s, s^2, s^3, ldots, s^i, s^{i + 1}, ldots tag 6$



                      must repeat itself at some point; that is,



                      $exists k, l in Bbb N, ; l ge k + 1, tag 7$



                      with



                      $s^l = s^k; tag 8$



                      then



                      $s^{l - k} = e; tag 9$



                      thus



                      $e = s^{l - k} in S; tag{10}$



                      it follows then that



                      $s^{l - k - 1}s = e tag{11}$



                      and clearly



                      $s^{l - k - 1} in S; tag{12}$



                      thus the group identity $e in S$, and every $s in S$ has an inverse in $S$; $S$ is thus a subgroup of $G$.






                      share|cite|improve this answer









                      $endgroup$


















                        2












                        $begingroup$

                        The proof presented here by our OP ForIgreaterthan_I appears to be logically valid; indeed, I like it because it is clever and innovative.



                        The way I would approach this, which alas is standard and not in fact overly clever, being as it is indeed rather obvious and straightforward, is as follows:



                        Let



                        $s in S; tag 1$



                        consider the powers



                        $s^i in S; tag 2$



                        since



                        $S subset G tag 3$



                        and



                        $vert G vert < infty, tag 4$



                        we have



                        $vert S vert < infty tag 5$



                        as well; thus the sequence



                        $s, s^2, s^3, ldots, s^i, s^{i + 1}, ldots tag 6$



                        must repeat itself at some point; that is,



                        $exists k, l in Bbb N, ; l ge k + 1, tag 7$



                        with



                        $s^l = s^k; tag 8$



                        then



                        $s^{l - k} = e; tag 9$



                        thus



                        $e = s^{l - k} in S; tag{10}$



                        it follows then that



                        $s^{l - k - 1}s = e tag{11}$



                        and clearly



                        $s^{l - k - 1} in S; tag{12}$



                        thus the group identity $e in S$, and every $s in S$ has an inverse in $S$; $S$ is thus a subgroup of $G$.






                        share|cite|improve this answer









                        $endgroup$
















                          2












                          2








                          2





                          $begingroup$

                          The proof presented here by our OP ForIgreaterthan_I appears to be logically valid; indeed, I like it because it is clever and innovative.



                          The way I would approach this, which alas is standard and not in fact overly clever, being as it is indeed rather obvious and straightforward, is as follows:



                          Let



                          $s in S; tag 1$



                          consider the powers



                          $s^i in S; tag 2$



                          since



                          $S subset G tag 3$



                          and



                          $vert G vert < infty, tag 4$



                          we have



                          $vert S vert < infty tag 5$



                          as well; thus the sequence



                          $s, s^2, s^3, ldots, s^i, s^{i + 1}, ldots tag 6$



                          must repeat itself at some point; that is,



                          $exists k, l in Bbb N, ; l ge k + 1, tag 7$



                          with



                          $s^l = s^k; tag 8$



                          then



                          $s^{l - k} = e; tag 9$



                          thus



                          $e = s^{l - k} in S; tag{10}$



                          it follows then that



                          $s^{l - k - 1}s = e tag{11}$



                          and clearly



                          $s^{l - k - 1} in S; tag{12}$



                          thus the group identity $e in S$, and every $s in S$ has an inverse in $S$; $S$ is thus a subgroup of $G$.






                          share|cite|improve this answer









                          $endgroup$



                          The proof presented here by our OP ForIgreaterthan_I appears to be logically valid; indeed, I like it because it is clever and innovative.



                          The way I would approach this, which alas is standard and not in fact overly clever, being as it is indeed rather obvious and straightforward, is as follows:



                          Let



                          $s in S; tag 1$



                          consider the powers



                          $s^i in S; tag 2$



                          since



                          $S subset G tag 3$



                          and



                          $vert G vert < infty, tag 4$



                          we have



                          $vert S vert < infty tag 5$



                          as well; thus the sequence



                          $s, s^2, s^3, ldots, s^i, s^{i + 1}, ldots tag 6$



                          must repeat itself at some point; that is,



                          $exists k, l in Bbb N, ; l ge k + 1, tag 7$



                          with



                          $s^l = s^k; tag 8$



                          then



                          $s^{l - k} = e; tag 9$



                          thus



                          $e = s^{l - k} in S; tag{10}$



                          it follows then that



                          $s^{l - k - 1}s = e tag{11}$



                          and clearly



                          $s^{l - k - 1} in S; tag{12}$



                          thus the group identity $e in S$, and every $s in S$ has an inverse in $S$; $S$ is thus a subgroup of $G$.







                          share|cite|improve this answer












                          share|cite|improve this answer



                          share|cite|improve this answer










                          answered 4 hours ago









                          Robert LewisRobert Lewis

                          46.9k23067




                          46.9k23067






















                              ForIgreaterthanJ is a new contributor. Be nice, and check out our Code of Conduct.










                              draft saved

                              draft discarded


















                              ForIgreaterthanJ is a new contributor. Be nice, and check out our Code of Conduct.













                              ForIgreaterthanJ is a new contributor. Be nice, and check out our Code of Conduct.












                              ForIgreaterthanJ is a new contributor. Be nice, and check out our Code of Conduct.
















                              Thanks for contributing an answer to Mathematics Stack Exchange!


                              • Please be sure to answer the question. Provide details and share your research!

                              But avoid



                              • Asking for help, clarification, or responding to other answers.

                              • Making statements based on opinion; back them up with references or personal experience.


                              Use MathJax to format equations. MathJax reference.


                              To learn more, see our tips on writing great answers.




                              draft saved


                              draft discarded














                              StackExchange.ready(
                              function () {
                              StackExchange.openid.initPostLogin('.new-post-login', 'https%3a%2f%2fmath.stackexchange.com%2fquestions%2f3123182%2fabstract-algebra-subgroup-proof-verification%23new-answer', 'question_page');
                              }
                              );

                              Post as a guest















                              Required, but never shown





















































                              Required, but never shown














                              Required, but never shown












                              Required, but never shown







                              Required, but never shown

































                              Required, but never shown














                              Required, but never shown












                              Required, but never shown







                              Required, but never shown







                              Popular posts from this blog

                              Szabolcs (Ungheria) Altri progetti | Menu di navigazione48°10′14.56″N 21°29′33.14″E /...

                              Discografia di Klaus Schulze Indice Album in studio | Album dal vivo | Singoli | Antologie | Colonne...

                              How to make inet_server_addr() return localhost in spite of ::1/128RETURN NEXT in Postgres FunctionConnect to...